| Hosted by CoCalc | Download
1
\documentclass{amsart}
2
\usepackage{sagetex}
3
\usepackage{amsmath,amssymb,nopageno,ulem}
4
\usepackage{mathtools}
5
\DeclarePairedDelimiter{\ceil}{\lceil}{\rceil}
6
%\usepackage[all]{xy}
7
\usepackage[shortlabels]{enumitem}
8
\renewcommand{\dot}{\bullet}
9
\newtheorem{define}{Definition}
10
\newtheorem{lemma}{Lemma}
11
\newtheorem{ex}{Exercise}
12
\newcommand{\aut}[1]{\mathbf{Aut}{#1}}
13
\newcommand{\conj}[2][g]{{#1}^{-1}{#2}{#1}}
14
\newcommand{\normal}{\triangleleft}
15
\newcommand{\Aut}{\operatorname{Aut}}
16
\newcommand{\Inn}{\operatorname{Inn}}
17
\newcommand{\Out}{\operatorname{Out}}
18
\newcommand{\gens}[1]{\left\langle{#1}\right\rangle}
19
\newcommand{\epi}{\twoheadrightarrow}
20
\newcommand{\epic}{\twoheadrightarrow}
21
\newcommand{\mono}{\hookrightarrow}
22
\newcommand{\Z}{\mathbf{Z}}
23
\newcommand{\C}{\mathbf{C}}
24
\newcommand{\R}{\mathbf{R}}
25
\newcommand{\F}{\mathbf{F}}
26
\newcommand{\N}{\mathbf{N}}
27
\newcommand{\Q}{\mathbf{Q}}
28
\newcommand{\Int}{\overset{\circ}}
29
\def\GL{\text{GL}}
30
\def\SL{\text{SL}}
31
\def\nin{\not\in}
32
\def\F{\mathbf{F}}
33
\def\ker{\text{ker}}
34
\def\Re{\mbox{Re}}
35
\def\inf{\mbox{inf}}
36
\def\sup{\mbox{sup}}
37
\def\ne{\not =}
38
\newcommand{\inv}[1]{{#1}^{-1}}
39
\newcommand{\ndef}{\overset{\text{def}}=}
40
\def\smat{[\begin{smallmatrix} b&c\\ a&b \end{smallmatrix}]}
41
\def\acts{\curvearrowright}
42
\newcommand{\legendre}[1]{\left(\frac{ #1 }{p}\right)}
43
\newcommand*\colvec[3][]{
44
\begin{pmatrix}\ifx\relax#1\relax\else#1\\\fi#2\\#3\end{pmatrix}
45
}
46
\renewcommand{\theenumi}{\alph{enumi}}
47
%%%%%%%%%%%%%%%%%%%%%%%%%%%%%%%%%%%%%%%%%%%%%%%%
48
\begin{document}
49
\section*{Ch2 Exercises}
50
\ex{Prove that the empty set is a subset of every set}
51
\begin{proof}
52
For $B \subseteq S \in \mathbf{Set}, B^c \cap B = \emptyset \subseteq
53
S$
54
\end{proof}
55
56
\ex{Prove the set of all algebraic numbers are countable. \\
57
Hint: For $\sum^n_0{a_k z^k} = 0, N \in \N$, there are only finitely many equations with \[n + \sum|a_k|=N\]
58
}
59
\begin{proof}
60
Write $\bold{A}$ for the algebraic numbers. Since each algebraic number is determined by its (finite) integral coefficients, then for some $n$ we have inclusions
61
\[A \mono \Z[X] \mono \Z^n, a \mapsto a_{n-1}x^{n-1}+...+a_0 \mapsto (a_0,...,a_{n-1})\]
62
Since $\Q \subset \bold{A}$, the algebraic numbers are countably infinite.
63
\end{proof}
64
65
\ex{Prove that there exist real numbers which are not algebraic.}
66
\begin{lemma}[Gelfond-Schneider Thm] $\alpha \ne 0,1, \beta \nin \Q, \alpha^\beta$ is transcendental.
67
\end{lemma}
68
\ex{Is the set of all irrational real numbers countable?}
69
\begin{proof}
70
Note there is an inclusion $\R/\Q \overset{\iota}\mono \R \setminus
71
\Q$ of irrational numbers with rational multiple exactly 1. If $\R/\Q$ is countable, $\R$ is a countable union of countable sets $\{xq\}_{q \in \Q}$ and therefore countable, which we know to be untrue. Therefore $\R/\Q$ is uncountable. Since $\iota(\R/\Q) \subset \R \setminus \Q$, the irrational numbers must also be uncountable.
72
\end{proof}
73
74
\ex{Construct a bounded set of real numbers with exactly three limit points.}
75
\begin{proof}
76
For $n \in \N$,
77
\[
78
A_0 \ndef \{1/n\}, A_1 \ndef \left\{\frac{n-1}{n}\right\}, A_2 \ndef
79
\left\{\frac{-n+1}{n}\right\}
80
\]
81
Let $A$ be the union of $A_0, A_1, A_2$, then $A \subset (-1,1)$ and
82
its limit points are exactly $\{\pm 1, 0\}$.
83
\end{proof}
84
\ex{Let $E'$ be the set of all limit points of a set $E$. Prove that $E'$ is closed. Prove that $E,\bar{E}$ have the same limit points. Do $E,E'$ always have the same limit points?}
85
For any limit point $\ell \in E'$, we have $\ell: N(\ell) \cap E'\subseteq E'; \ne \emptyset$
86
\ex{$A_1,A_2,A_3,\dots$ subsets of a metric space. \\
87
\begin{enumerate}
88
\item For $B_n \ndef \bigcup^n A_i$, show $\bar{B_n} = \bigcup^n \bar{A_i}$
89
\item For $B \ndef \bigcup^\Z A_i$, show $\bar{B} \subset \bigcup \bar{A_i}$
90
\end{enumerate} Show this inclusion can be proper.}
91
\begin{enumerate}
92
\item Consider $ n=2: \bar{A}_1 \cup \bar{A}_2$
93
\end{enumerate}
94
95
\ex{Are points of each open set $E \subseteq \R^2$ limit points? For closed sets?}
96
97
For $x \in E, \exists U_x \subset E$, its limit points would be $\ell:N(\ell) \cap E \setminus \{\ell\} \ne \emptyset$.
98
\ex{Let $\Int E$ denote the interior of $E$.
99
\begin{enumerate}
100
\item Prove $\Int E$ open
101
\item Prove $E$ open iff $\Int E = E$
102
\item For $G$ open, $G \subseteq E$, prove $G \subseteq \Int E$
103
\item Prove $(\Int E)^c = \bar{E^c}$
104
\item Do $E,\bar E $ always have the same interiors?
105
\item Do $E, \Int E$ always have the same closures?
106
\end{enumerate}
107
}
108
\begin{enumerate}
109
\item By definition, $A$ is open if for each point $p, \exists N(p)\subseteq A$, and the interior of $A$ is exactly such a set.
110
\item
111
\end{enumerate}
112
113
\ex{Let $X$ be infinite. For $p,q \in X$, define
114
\[
115
d(p,q) \ndef
116
\begin{cases}
117
1 & p \ne q \\
118
0 & p=q
119
\end{cases}
120
\]
121
Prove this is a metric. What are its open, closed and compact subsets?}
122
\begin{proof}
123
$x=y \iff d(x,y) = 0$ so positive definite; if $x \ne y, y \ne x \implies d(x,y) = d(y,x)$ so it's symmetric. To show the triangle inequality holds, break the right-hand side into cases:
124
\[
125
d(p,r)+d(r,q) =
126
\begin{cases}
127
2 & p \ne r \text{ and } r \ne q \\
128
1 & p=r \ne q \text{ or } p \ne r = q \\
129
0 & p=r=q
130
\end{cases}
131
\]
132
Then for $q,r,p$ pairwise distinct the sum (RHS) is always greater. Otherwise both sides equal $1$ (or $0$ if all terms equal).
133
134
Neighborhoods can't have zero radius, therefore $N(p) = \{ q:d(p,q) = 0\} = \{p\}, N(p) \cap A \subseteq \{p\}$ so no limit points, i.e., $A = \bar{A}$. Since $p \in A \implies N(p) \subseteq A, A = \overset{\circ}A$, and all subsets are clopen.
135
136
Let $\mathcal{U}$ be a cover of $A$. If $A$ is a finite subset with $n$ elements, we can finitely cover $A$ with
137
$\{U_k \in \mathcal{U}:x_k \in U_k\}_{j:1\leq j \leq n}$
138
, i.e., (finite) open sets each containing a point of $A$. Otherwise, for uncountable $A$, consider the cover $\mathcal{U}'\ndef \{U_j = x_j\}$. Any finite subcovering contains only finite points of $A$.
139
140
Thus, finite sets are compact and infinite sets noncompact.
141
\end{proof}
142
143
\ex{For $x,y \in \R$, define
144
\begin{itemize}
145
\item $d_1 \ndef (x-y)^2$
146
\item $d_2 \ndef \sqrt{|x-y|}$
147
\item $d_3 \ndef |x^2 - y^2|$
148
\item $d_4 \ndef |x-2y| $
149
\item $d_5 \ndef \frac{|x-y|}{1+|x-y|}$
150
\end{itemize}
151
Which are metrics?}
152
%\begin{proof}
153
Immediately, $d_4$ isn't symmetric [$d(1,4) = 7 \ne 2 = d(4,1)$] and
154
$d_3$ isn't positive definite [let $y = -x$]. Then of the remaining three we check whether or not we have the triangle inequality $d(x,z)+d(z,y)-d(x,y) \geq 0$:
155
\begin{itemize}
156
\item $(x-z)^2+(z-y)^2 -(x-y)^2= x^2 + y^2 +2z^2 -2z(x+y)-x^2-y^2-2xy = z^2 + (z-(x+y))^2 + (x+y)-2xy$
157
\item $$\sqrt{|x-z|} + \sqrt{|z-y|} = \frac{|x-z|-|z-y|}{\sqrt{|x-z|} - \sqrt{|z-y|}}$$
158
\item $$\frac{|x-z|}{1+|x-z|} + \frac{|z-y|}{1+|z-y|}$$
159
\end{itemize}
160
161
%\end{proof}
162
163
\ex[Exercise 12]{Prove $K\ndef \{0\} \cup \{1/n\}_{n \in \N}$ is compact directly from definition.}
164
\begin{proof}
165
Let $U$ be a open cover of $K$. For each $x_n\in K, \exists U_n \in U:x_n \in U_n$. Define $U_0:0 \in U_0$ to be the open set containing $0$. For each $\delta >0, \exists m \in \N: \{\frac{1}{m},\frac{1}{m+1},...\} \subset U_0 \cap N_\delta$. Thus, there are finitely many $U_n:n<m$ which, in addition to $U_0$, cover $K$.
166
\end{proof}
167
\ex{Construct a compact set of reals whose limit points are countable.}
168
%\begin{proof}
169
Something like Cantor set?
170
\[f(x) \ndef
171
\begin{cases}
172
1/r & x \in \Q \\
173
0 & x \nin \Q
174
\end{cases}
175
\]
176
%\end{proof}
177
178
\ex[14]{Give an example of an open cover of $(0,1)$ without finite subcover.}
179
\begin{proof}
180
Let $U \ndef \{(\frac{1}{n},1)\}_{n \in \N}$. Then for any $x \in (0,1), \exists M \in \N: x<1 \implies \frac{1}{M} < x$. So $U$ is a cover of $(0,1)$. However for any finite subset $U' \ndef \{(\frac{1}{n},1)\}_{n:1 \leq n \leq k } $ of $U$ with $k < M, \frac{1}{M} \nin U'$. Therefore $(0,1)$ is not compact.
181
\end{proof}
182
183
\ex[15]{Show that Thm 2.36 and its Corollary become false if compact is replaced with either closed or bounded.}
184
\ex[16]{$E \ndef \{p \in \Q:p^2 \in (2,3)\}$. Show $E$ is closed and bounded in $\Q$, but not compact. Is $E$ open?}
185
\begin{proof}
186
$1^2 < p^2 < 2^2 \implies E \subset (1,2)$, so $E$ bounded.
187
188
Assume $E$ is not closed, and note $\Q \setminus E = ub(E) \cup
189
lb(E)$. Say $E$ has a limit point $\ell \in
190
ub(E):(\ell-\delta,\ell+\delta) \cap E \ne \emptyset$. Then for some
191
$\ell' \in ub(E): \ell > \ell'$, set $\delta = \ell-\ell'>0$. Then we have
192
$(\ell-\delta,\ell+\delta) = (\ell',2\ell-\ell')$ and $
193
(\ell',2\ell-\ell') \cap E \ne \emptyset$, which is impossible since
194
$\ell'>p \in E$. A similar argument follows for $\ell \in lb(E)$,
195
thus $E$ is closed.
196
197
The cover $U \ndef \{(0,r- \frac{1}{n}):n \in \N, r^2 \in (2,3)\}$
198
has no finite subcover, so $E$ is not compact.
199
\end{proof}
200
201
202
\ex{Decimal expansions on the unit interval with only 4's and 7's
203
\[
204
E \ndef \left\{\sum a_k 10^{-k}:a_k \in \{4,7\}\right\}
205
\]
206
Is $E$ countable?dense?compact?perfect?}
207
\begin{proof}
208
There are really two cases here: series including eventually zero terms like $0.44=0.44\bar{0}$, and nonterminating series without zeros after the decimal point.
209
210
To determine cardinality, we map into sets of known cardinality. In the case including terminating series, $E$ is in surjection with binary decimals covering the unit interval $\sum a_k 10^{-k} \mapsto \sum b_k 2^{-k}$, otherwise there is a bijection in the latter case. So $E$ is uncountable either way.
211
212
$E$ is not dense in the unit interval because there are no terms of $E$ in the interval $(0.5,0.6)$.
213
214
We first show $E$ is perfect. Assume contrary, then there
215
exists $\ell \nin E:N_\epsilon(\ell) \cap E \ne \emptyset$. Then
216
$\ell$ has a decimal expansion with digits which aren't 4,7, or in
217
certain cases, 0 [for example, $0.404$]. If the first such digit
218
occurs at $k=n$, then for any $s \in E, |\ell -
219
s|>10^{-m-1}$. Therefore $\ell$ cannot be a limit point, and $E$ is
220
perfect.
221
222
Since $E$ is perfect, it contains its limit points and is therefore closed. Either $(0.4,0.7)$ or $(0.\bar{4},0.\bar{7}) = (\frac{4}{9}, \frac{7}{9})$ is a bound for $E$, and since $E$ is closed and bounded, $E$ is compact.
223
\end{proof}
224
225
\ex[18]{Is there a nonempty perfect set in $\R$ which contains no rational number?}
226
Assume there is such a set $P$, then for $q \in \Q, \epsilon > 0$, the intersection
227
\[
228
N_\epsilon(q) \cap (P \setminus \{q\})
229
\]
230
must be empty.
231
232
Since $P$ is nonempty, we have $p \in P:p \in (p - \epsilon, p+ \epsilon)$. We show that any open interval $(a,b)$ contains rational points. [WTS open segment has rationals]
233
\ex[19]{Prove:
234
\begin{enumerate}
235
\item Disjoint closed sets $A,B$ in a metric space $X$ are separated
236
\item Prove the same for disjoint open sets.
237
\item For some $p\in X, A \ndef \{q:d(p,q)<\delta\}, B \ndef \{q:d(p,q)>\delta\}$. $A,B$ are separated.
238
\item Every connected metric space with at least two points is uncountable.
239
\end{enumerate}
240
}
241
\begin{enumerate}
242
\item For disjoint closed sets, $A \cap B = \emptyset, A = \bar{A}, B = \bar{B}$. Then $\bar{A} \cap \bar{B} = \emptyset$
243
\item For disjoint open subsets $A,B$, $A \cup B$ also open
244
\end{enumerate}
245
\ex[20]{Are closures and interiors of connected sets connected?}
246
\ex[21]{Let $A,B \subset \R^k$ be separated subsets. Then for $a \in A, b \in B, t \in \R$, define
247
\[
248
p(t) = (1-t)a+tb, A_o=\inv{p}(A), B_0=\inv{p}(B)
249
\]
250
Prove
251
\begin{enumerate}
252
\item $A_0,B_0$ are separated subsets.
253
\item $\exists t_0 \in (0,1):p(t_o)\nin A \cup B$
254
\item Every convex subset of $\R^k$ is connected.
255
\end{enumerate}
256
}
257
Rewrite as $p(t) = a+ t(b-a)$. On the closed unit interval, $p(t)$ deforms $a$ into $b$ and vice-versa.
258
\ex[22]{A metric space is \emph{separable} if it contains a countable dense subset. Prove $\R^k$ is separable.}
259
%\begin{proof}
260
TBD:Show cart product of intervals interior $N_\delta(x) \subset \R^k$
261
262
For $x = (x_1,...,x_k) \in \R^k, E \ndef \{(q_i,q_j)_{i,j:1 \leq i,j \leq k} \subseteq \Q^k:q_i,q_j \in \Q\}$, the cartesian product of open intervals, we will show $N_\delta(x) \cap E$ is an open subset of $\Q^k$ and $\R^k$ is thus separable. \\
263
For each \( \delta >0, \exists n \in \N:1/n < \delta \). Then
264
$B_{1/n}(x) \subset B_\delta (x)$.
265
%\end{proof}
266
267
\ex[23]{Prove that every separable metric space has a countable base.}
268
\ex[24]{Let $X$ be a metric space in which every infinite subset has a limit point. Prove $X$ is separable.}
269
\ex[25]{Prove every compact metric space $K$ has countable base, and that $K$ is therefore separable.}
270
\ex[26]{Let $X$ be a metric space in which every infinite subset has a limit point. Prove $X$ is compact.}
271
\ex[27]{For $E\subseteq \R^k$ uncountable and $P$ its condensation points, prove $P$ is perfect and $P^c \cap E$ at most countable.}
272
\ex[28]{Prove that every closed set in a separable metric space is the union of a (perhaps empty) perfect set and an at most countable set. (corollary: every countable closed subset of $\R^k$ has isolated points) hint: ex27}
273
\ex[29]{Prove that open sets in $\R$ are the union of at most countable intervals.}
274
Define a relation on $U \subseteq \R$ as follows: $\forall x,y \in U, x \sim y \iff $ there is a closed interval of $U$ with endpoints $x,y$. This relation is reflexive and symmetric, and for $x \sim z, z \sim y$, the union of closed segments is again in $U$.
275
276
277
278
\ex[30]{Imitate the proof of Thm 2.43 to obtain the following result: \\
279
If $\R^k = \bigcup_1^\infty F_n$, where each $F_n$ is a closed subset then at least one $F_n$ has nonempty interior. \\
280
Equivalent statement: If $G_n$ is a dense open subset of $\R^n$, then
281
$\bigcap_1^\infty G_n \ne \emptyset$ (in fact, dense in $\R^n$).}
282
\newpage
283
\section*{definitions and results}
284
\begin{itemize}
285
\item [limit point] We say $\ell$ is a \emph{limit point} of $E$ if there exists $\{q:q \ne \ell\} \subset E \cap N_r(\ell)$
286
\item [closure] The closure $\bar{A}$ of $A \subseteq X$ is $A$ plus its limit points.
287
\item [compact] A subset $S \subseteq X$ is \emph{compact} if each cover $\{U_i\}_I$ admits a finite subindexing $\{U_i\}_{J\subseteq I}$, i.e., a finite subcover.
288
\item [perfect] A subset is \emph{perfect} if it is closed and each point is a limit point.
289
\item [dense] $A$ is \emph{dense} in $X$ if for any other subset $B, B \cap \bar{A} \ne \emptyset$.
290
\item [condensation point] A point $p$ such that $|N_r(p) \cap E| = \mathfrak{c}$
291
\item [separate] $A,B$ are \emph{separated} if $A \cap \bar{B} = \emptyset \text{ and } \bar{A} \cap B = \emptyset$
292
\item [connected] $S$ is \emph{connected} if it is not the disjoint union of separable sets.
293
\item [base] A collection of open subsets $\{V_a\}$ such that every open set is some union $\bigcup V_a$.
294
\end{itemize}
295
We want to show there exists a nontrivial open subset of $\Q^k$ interior to every open subset of $\R^k$. \\
296
Assuming Euclidean metric, set $\delta = \frac{\sqrt{n}}{n^2}\epsilon$ and define $\Delta \ndef (\delta,...,\delta) \in \R^k$.
297
\[
298
d(x, x+\Delta) = \sqrt{\sum{\left(x_i-(x_i+\frac{\sqrt{n}}{n^2}\epsilon)\right)^2}} = \frac{\epsilon}{n} \in N_\epsilon(x)
299
\]
300
By the archimidean property of $\R$, there exists $M \in \N, M>1$:
301
\[
302
\frac{\sqrt{n}}{n^2}\epsilon<1 \implies \inv{M} < \frac{\sqrt{n}}{n^2}\epsilon
303
\]
304
Therefore, we have the open subset $E \subset \Q^k$:
305
\[
306
E \ndef \Q^k \cap \prod_{i=1}^n(x_i-\delta,x_i+\delta) \subset \prod_{i=1}^n(x_i-\delta,x_i+\delta) \subset N_\epsilon(x)
307
\]
308
Moreover for $(\inv{M},...,\inv{M}) \in N_\epsilon(0)$, so $E$ is nonempty. [show $N_\epsilon(x) \subset U(x)$] \\
309
310
WTS (countable) rationals in any real open set. Any real open set is the disjoint product of at most countable open intervals. Since $\R \cong (0,1)$, it suffices to prove this on the unit interval. Let $(a,b) \subset (0,1)$. Then $0 < \frac{1}{M} < b-a<1, M \in \N$.
311
312
From this we can show $b-\inv{M}>a, a+\inv{M}<b \implies a+\inv{M} -b+\inv{M} < b-a, -(b-a) + 2 $
313
For any $x \in (a,b), x-a,b-x>0, 2x+b-a>0$
314
WTS $\exists \delta>0:N_\delta(x) \cap U_x \ne emptyset$ \\
315
316
\[
317
\sage{latex(continued_fraction(pi).str(nterms=2))}
318
\]
319
320
\section*{things which are actually wrong}
321
Let $U,L$ be the open upper and open lower halves of $\R^k$, i.e., given $x = (x_1,...,x_n),(0,...,0) \in \R^k,
322
\begin{cases}
323
x_1 > 0 \implies & x \in U \\
324
x_1 < 0 \implies & x \in L \\
325
x_1 = 0 \implies & x \nin U \cup L
326
\end{cases}$. Then for $\delta >0, (\pm \delta,0,...,0) \in N_r(0) \cap (A \subset U) N_r(0) \cap (B \subset L), 0 \in \bar{U}\cap\bar{L}$ therefore by definition $(U \cap \bar{L}) \cap (\bar{U} \cap L) = \emptyset$.
327
We will show these definitions are equivalent.
328
\end{document}
329
330
331